What is the Factored Form of the Derivative Using Product Rule?

  • Thread starter Thread starter cataschok
  • Start date Start date
  • Tags Tags
    Derivative Form
Click For Summary
To find the factored form of the derivative of g(x)=(2x+1)^2(x-7)^3 using the product rule, the initial derivative is calculated as 4(2x+1)(x-7)^3 + 3(2x+1)^2(x-7)^2. The next step involves factoring out common elements, specifically (2x+1)(x-7)^2. By simplifying the expression within the brackets, the final factored form is achieved as 5(x-7)^2(2x-5)(2x+1). This method effectively consolidates the derivative into a more manageable format.
cataschok
Messages
6
Reaction score
0

Homework Statement


use product rule to find the derivative of g(x)=(2x+1)^2(x-7)^3


Homework Equations


i've applied chain rule and product rule to get...


The Attempt at a Solution


4(2x+1)(x-7)^3+3(2x+1)^2(x-7)^2

i need to factor out to get the following..
5(x-7)^2(2x-5)(2x+1)
That's the final answer from the book. But the steps to get there... I'm lost. Can anyone help me out on getting it to the factored form?
 
Physics news on Phys.org
cataschok said:

Homework Statement


use product rule to find the derivative of g(x)=(2x+1)^2(x-7)^3


Homework Equations


i've applied chain rule and product rule to get...


The Attempt at a Solution


4(2x+1)(x-7)^3+3(2x+1)^2(x-7)^2

i need to factor out to get the following..
5(x-7)^2(2x-5)(2x+1)
That's the final answer from the book. But the steps to get there... I'm lost. Can anyone help me out on getting it to the factored form?
Look for factors that are common to both terms.
4(2x+1)(x-7)3+3(2x+1)2(x-7)^2
= (2x+1)(x - 7)2[4(x - 7) + 3(2x + 1)]

Now combine the terms in the brackets and pull out the common factor there, and you'll have what you need.
 
thank you kind sir, you have saved me many hours of frustration!
 
Question: A clock's minute hand has length 4 and its hour hand has length 3. What is the distance between the tips at the moment when it is increasing most rapidly?(Putnam Exam Question) Answer: Making assumption that both the hands moves at constant angular velocities, the answer is ## \sqrt{7} .## But don't you think this assumption is somewhat doubtful and wrong?

Similar threads

Replies
15
Views
3K
  • · Replies 6 ·
Replies
6
Views
2K
  • · Replies 18 ·
Replies
18
Views
2K
  • · Replies 7 ·
Replies
7
Views
2K
  • · Replies 9 ·
Replies
9
Views
2K
  • · Replies 10 ·
Replies
10
Views
2K
  • · Replies 25 ·
Replies
25
Views
2K
  • · Replies 4 ·
Replies
4
Views
2K
  • · Replies 3 ·
Replies
3
Views
2K
  • · Replies 5 ·
Replies
5
Views
2K